Fråga:
Likställer $ \ frac {dp} {dt} $ och $ F $ vid ett hydrauliskt hopp?
MadCommy
2015-09-21 16:22:14 UTC
view on stackexchange narkive permalink

En tidskrift jag läser om cirkulära hydrauliska hopp ger ekvationen och jag citerar:

$ H $ Djup efter hydrauliskt hopp , $ h $ djup före hydrauliskt hopp, $ V $ hastighet efter hydrauliskt hopp, $ v $ hastighet före hydrauliskt hopp, $ R $ radie för hydrauliskt hopp.

Villkoret för bevarande av momentum kan skrivas som $$ \ frac {dp} {dt} = 2 \ pi R \ rho HV ^ 2 - 2 \ pi R \ rho hv ^ 2 = F_1-F_2 $$ där $$ F_1 = 2 \ pi R \ rho g \ int_0 ^ hx \ cdot dx = \ rho g \ pi Rh ^ 2 $$ $$ F_2 = 2 \ pi R \ rho g \ int_0 ^ Hx \ cdot dx = \ rho g \ pi RH ^ 2 $$

Jag förstår att enheter för $ F $ och $ \ frac {dp} {dt} $ span > är båda $ kg \ m \ s ^ {- 2} $ , och att de är likvärdiga.

Men jag vet inte hur, i detta exempel, $ F_1 $ och $ F_2 $ härleds dvs där $ g $ och integralerna kommer från . Kan någon snälla förklara?

Sida 4 från: På det cirkulära hydrauliska hoppet

Korspost till fysik: http://physics.stackexchange.com/questions/208249/equating-fracdpdt-and-f
Ett svar:
Chris Mueller
2015-09-21 17:42:25 UTC
view on stackexchange narkive permalink

$ F = \ frac {dp} {dt} $ är i viss mening ett mer grundläggande uttryck för Newtons lag än $ F = ma $ eftersom $ F = ma $ inte tillåter situationer med massändring. Du kan enkelt hämta den andra från den förra genom att använda $ p = mv $ vilket ger $ F = m \ frac {dv} {dt} = ma $.

Din fråga verkar handla mer om hur man härleder uttrycken för kraften vid gränsen till det hydrauliska hoppet. För att göra detta, tänk på det hydrauliska hoppet som en vägg; på ena sidan av väggen är vattenhöjden $ h $ och på den andra är det $ H $.

Tänk nu på ett oändligt stort kvadrat i denna vägg på djup $ x $; trycket som utövas på denna kvadrat ges av $$ P = \ rho \ g \ x. $$ Kraften som utövas på kvadraten beräknas genom att multiplicera med ytan på det oändliga plåstret $$ F_ {inf} = \ rho \ g \ x \ dx \ dy. $$ Nu behöver du bara summera (med integration) alla plåster på väggen. Y-riktningen är enkel $$ \ begin {align} F_ {tot} & = \ rho \ g \ int_ {0} ^ {h} \ left (\ int_0 ^ {2 \ pi R} dy \ right) xdx \\ & = \ rho \ g \ 2 \ pi R \ int_0 ^ {h} x dx \\ & = \ pi R \ \ rho \ g \ h ^ 2 \ end {align} $$

enter image description here

Tack för den enastående tydliga förklaringen! Jag hade fel antagit att ekvationerna för $ F_ {tot} $ härleddes från momentum bevarande ekvationen; men istället är de helt separata. Har du något emot att återskapa det här svaret för min identiska fråga på webbplatsen för fysikstackexchange (som ännu inte har gett så bra svar)?
@MadCommy Jag är glad att du tyckte att mitt svar var till hjälp, du kan [acceptera] (http://engineering.stackexchange.com/help/accepted-answer) det om du känner att det helt besvarade din fråga. Korsposterande frågor till flera olika webbplatser är vanligtvis missnöjda eftersom det slutar fragmentera frågan. Jag kommer inte att lägga om mitt svar på Physics.SE, men jag har lagt till länkar till båda frågorna så att andra är medvetna om båda. I framtiden kan du försöka välja lämplig webbplats och bara ställa en enda fråga.
Tack för rådet; Jag var inte säker på om min fråga var mer lämpad för fysik eller teknik eftersom jag, även om jag var ett tekniskt ämne, undersökte den för min avancerade uppsats i fysik.
Inga problem. Jag skulle säga att frågor om vätskemekanik i allmänhet tas emot bättre här på Engineering.SE. Fysiker studerar överraskande inte vätskemekanik som en del av standardplanen.
Varför är både $ V $ och $ v $ oberoende av djupet $ x $ medan trycket är?


Denna fråga och svar översattes automatiskt från det engelska språket.Det ursprungliga innehållet finns tillgängligt på stackexchange, vilket vi tackar för cc by-sa 3.0-licensen som det distribueras under.
Loading...